Select the FIRST correct reason why the given series converges.

  • Thread starter Thread starter McAfee
  • Start date Start date
  • Tags Tags
    Reason Series
Click For Summary
The discussion focuses on identifying the correct reason for the convergence of a given series. Participants evaluate various convergence tests, including geometric series, p series, comparison tests, and the alternating series test. The consensus leans towards the alternating series test as the primary reason for convergence, noting that the series alternates, decreases in absolute value, and approaches zero as n approaches infinity. Some participants also consider geometric series and comparison tests but ultimately emphasize the alternating series test. The thread concludes with a participant successfully determining all answers after further investigation.
McAfee
Messages
96
Reaction score
1

Homework Statement



Select the FIRST correct reason why the given series converges.

A. Convergent geometric series
B. Convergent p series
C. Comparison (or Limit Comparison) with a geometric or p series
D. Converges by alternating series test

XVOO5.jpg


The Attempt at a Solution


I will go over my reasoning for each problem.

1. A. I really it was a geometric and r is 1/16 which is less than 1 meaning it is convergent.

2. I want to say it is comparsion but I'm not sure.

3. Not sure.

4. B. Want to say alternating but it doesn't fit.

5. D. It alternates, ignoring signs it decrease, and the limit as x goes to infinity equals 0. All meaning it converges due to the alternating series test.

6. D. It alternates, ignoring signs it decrease, and the limit as x goes to infinity equals 0. All meaning it converges due to the alternating series test.
 
Physics news on Phys.org
If anyone has any ideas. I'm truly stuck on this one problem.
 
Just to update everyone. After further investigation I was able to figure out all the answers.
Here they are.

1. A. I really it was a geometric and r is 1/16 which is less than 1 meaning it is convergent.

2. B. Rewrite this as (-1)^n * n / (n^9 * (-1)^n) = 1/n^8.

3. C. Use 0 < sin^2(3n) < 1, so that sin^2(3n)/n^2 < 1/n^2

4. D. It alternates, ignoring signs it decrease, and the limit as x goes to infinity equals 0. All meaning it converges due to the alternating series test.

5. D. It alternates, ignoring signs it decrease, and the limit as x goes to infinity equals 0. All meaning it converges due to the alternating series test.

6. D. It alternates, ignoring signs it decrease, and the limit as x goes to infinity equals 0. All meaning it converges due to the alternating series test.
 
Question: A clock's minute hand has length 4 and its hour hand has length 3. What is the distance between the tips at the moment when it is increasing most rapidly?(Putnam Exam Question) Answer: Making assumption that both the hands moves at constant angular velocities, the answer is ## \sqrt{7} .## But don't you think this assumption is somewhat doubtful and wrong?

Similar threads

  • · Replies 3 ·
Replies
3
Views
1K
  • · Replies 1 ·
Replies
1
Views
2K
  • · Replies 7 ·
Replies
7
Views
2K
  • · Replies 5 ·
Replies
5
Views
2K
  • · Replies 6 ·
Replies
6
Views
2K
  • · Replies 14 ·
Replies
14
Views
2K
  • · Replies 11 ·
Replies
11
Views
3K
  • · Replies 2 ·
Replies
2
Views
2K
  • · Replies 3 ·
Replies
3
Views
1K
Replies
14
Views
2K